If you're seeing this message, it means we're having trouble loading external resources on our website.

If you're behind a web filter, please make sure that the domains *.kastatic.org and *.kasandbox.org are unblocked.

Main content

Course: LSAT > Unit 1

Lesson 5: Analytical Reasoning – Worked examples

Ordering setup | New info–could be true 1 | Worked example

Watch a demonstration of one way to approach a "New info: could be true" question on an ordering setup from the analytical reasoning section of the LSAT.

Want to join the conversation?

Video transcript

- [Instructor] Make sure that you've already watched the setup video in which we created the initial diagram that you'll see us using here. So the question asks if S was begun in a later year than L, which one of the following could be true? We are looking for the choice that's possible given the new and temporary information that S is later than L. So since we're given new information that isn't necessarily true outside of this particular question, let's redraw our initial diagrams that we can work from it and make deductions that are specific to this scenario OE. All right, now this new rule doesn't tell us anything specific about placement, unfortunately. It only tells us that S is later than L. So what can we do? We have our big chain of elements here. And we said that initially we were only missing S from this chain. So now that we can add in S, our chain will account for every monument and we should be able to make some deductions now. With S being after L, that means that S can't be part of the pair in 601. So that means that G and M must be the monuments that are first. L is the only monument that can be second in order to leave room for S, F and H to be later than L in some order. And then since H can only be in 604, 605, that means that F or S is the monument in 603. We can't determine anything else for certain. So let's find the choice that could be true. A. F was begun in 604. This is possible but I know it can be scary when A seems like the answer so let's do a quick test. If F is in four, H would be in five. F has to be later than L in this question so S would be third, L would have to be second and then G M would be first. This works. Let's quickly go over the wrong choices. G was begun in 602. This must be false because G is in 601 here. C. L was begun in 603. This is also false because L is in 602 here. D. M was begun in 603. This can't be possible because M is in 601. And E. S was begun in 602. This can't happen in this question because we deduced that S has to be third, fourth, or fifth. Great work on this question. The answer is A.